All Problems

Laws of Conservation

Problem 1.175

A particle of mass \(m_{1}\) collides elastically with a stationary particle of mass \(m_{2}\left(m_{1}>m_{2}\right)\). Find the maximum angle through which the striking particle may deviate as a result of the collision.

Reveal Answer
sinθmax=m2/m1\sin \theta_{\max }=m_{\mathbf{2}} / m_{1}